PSAT Reading Question 305: Answer and Explanation

Question: 305

Which choice from Passage 2 provides the best evidence for the answer to the previous question?

  • A. Lines 55–59 ("An experiment…mass")
  • B. Lines 63–65 ("This measurement…eV")
  • C. Lines 72–73 ("During this…neutrino")
  • D. Line 74 ("KATRIN cannot…directly")

Correct Answer: C

Explanation:

C

(See explanation above.)

Test Information

  • Use your browser's back button to return to your test results.
  • Do more Reading tests.

More Tests

    All content of site and practice tests © 2022 Jack.
    Quick View

    PSAT Practice Tests

    More Information